9
$\begingroup$

Let $ f:A\to B$ be an injective, local homorphism between two Noetherian local rings. Consider the completions $\hat A$ and $\hat B$ with respect the maximal ideals. We have an induced homomorphism $\hat f: \hat A \to\hat B$. What assumptions do we need in order to ensure that also $\hat f$ is injective?

My main interest is geometric, so for example the local map induced by a surjective morphism between Noetherian schemes.

Many thanks in advance

$\endgroup$
5
  • 1
    $\begingroup$ Trivia: if $f$ is an homeomorphism onto its image (e.g., $A$ and $B$ are DVR), so is $\hat{f}$. $\endgroup$
    – Luc Guyot
    Jun 2, 2018 at 15:01
  • $\begingroup$ Uhm, yes I'm interested in the DVR case. But why? $\endgroup$
    – manifold
    Jun 2, 2018 at 15:21
  • 1
    $\begingroup$ Assume that $A$ and $B$ are DVRs with uniformizing elements $\pi_A \in A$ and $\pi_B \in B$. Write $f(\pi_A) = u \pi_B^{\nu}$ with $u$ a unit of $B$ and $\nu$ a positive integer. Then $f(A) \cap \pi_B^{n \nu} B \subseteq f(\pi_A^{n}A)$ for every $n \ge 0$, hence $f$ is open. $\endgroup$
    – Luc Guyot
    Jun 2, 2018 at 18:46
  • $\begingroup$ The natural condition is that the topology on $A$ induced by powers of its maximal ideal and the topology induced by the powers of the maximal ideal of $B$ intersected with $A$ are the same. $\endgroup$
    – Mohan
    Jun 3, 2018 at 20:41
  • $\begingroup$ Trivia continued: If $\hat{f}$ is injective and if $\hat{A}$ is compact (i.e, the residue field of $A$ is finite), then $f$ is an homeomorphism onto its image. So the "natural condition" is necessary under the compactness assumption. $\endgroup$
    – Luc Guyot
    Jun 4, 2018 at 20:19

1 Answer 1

5
$\begingroup$

This may be useful:

Proposition (Zariski) (see EGA I, (3.9.8) in Springer edition)

Let $f: (A,\mathfrak{m})\to (B,\mathfrak{n})$ be a local homomorphism of noetherian local rings. Assume that:

  • $f$ is injective.
  • $\hat{A}$ is a domain.
  • $f$ is essentially of finite type.

Then the $\mathfrak{m}$-adic topology on $A$ is induced by the $\mathfrak{n}$-adic topology on $B$.

Of course this implies that $\hat{f}$ is injective. If we don't assume that $\hat{A}$ is a domain (but $A$ is) it is easy to construct counterexamples.

$\endgroup$
4
  • $\begingroup$ Wait, but what about the following example: $A=\mathbb Z[t]_{(p,t)}$ and $B=\mathbb Z[t]_{(t)}$. Clearly $A\subset B$, but $\hat A= \mathbb Z_p[[t]]$ and $\hat B=\mathbb Q[[t]]$ $\endgroup$
    – manifold
    Jul 18, 2018 at 19:54
  • $\begingroup$ Moreover I'm looking at EGA I (chapter 0 of Springer edition), and I cannot find any 3.9.8. $\endgroup$
    – manifold
    Jul 18, 2018 at 20:04
  • 1
    $\begingroup$ @manifold: your example is not a local homomorphism (the map $A_{\mathfrak p} \to A_{\mathfrak q}$ for $\mathfrak p \supsetneq \mathfrak q$ never is). $\endgroup$ Jul 18, 2018 at 21:41
  • $\begingroup$ @manifold: it's 3.9.8 of chapter 1 (not chapter 0), on page 255. $\endgroup$ Jul 19, 2018 at 7:25

Your Answer

By clicking “Post Your Answer”, you agree to our terms of service and acknowledge you have read our privacy policy.

Not the answer you're looking for? Browse other questions tagged or ask your own question.